Talk:2014 AMC 12A Problems/Problem 15

Revision as of 10:00, 3 March 2015 by Hesa57 (talk | contribs) (Created page with "== Unusable Solution == I deleted the solution that listed out every possible number and added them up. This problem is from the AMC 12, and doing so is not a viable solution.")
(diff) ← Older revision | Latest revision (diff) | Newer revision → (diff)

Unusable Solution

I deleted the solution that listed out every possible number and added them up. This problem is from the AMC 12, and doing so is not a viable solution.